"MSS has wiggle room." How much? Forum

Prepare for the LSAT or discuss it with others in this forum.
Post Reply
Kimikho

Gold
Posts: 3971
Joined: Fri Mar 16, 2012 7:01 pm

"MSS has wiggle room." How much?

Post by Kimikho » Sat Sep 21, 2013 1:19 am

I don't have my books until Monday, but since I've read them all and am still having trouble, I'm still going to post this so it doesn't drive me insane all weekend.

I get the difference between MSS and MTB, but I'm not getting, one, the strategy for answering these, and two, the entire premise (that MSS questions are "more" supported but not "entirely" supported) seems to me to contradict the whole "there is only one right answer and four wrong answers."

FOR EXAMPLE here is my thought process through PT 68 #2 S4 that I just missed :oops: :

A.) Well, this is just here to trip us up. Just because mice show up more doesn't mean they don't show up at all in unfragged forests.
B.) "Some animals, such as white-footed mice" =/= most species of small animals.
C.) This isn't stated anywhere at all.
D.) Well, lyme disease sucks and cutting forests this way can increase the carriers of the bacteria. I guess this could be correct. I mean it has wiggle room but it could be correct too...
E.) Well, the stim says "reaching their highest population..." and that they are the main carrier of deer ticks...well then...

The answer is D.

BASICALLY my problem is that most guides either don't split up MSS/MTB, or do and don't provide a roadmap to answering them, and saying "MSS have wiggle room" confuses me. How much wiggle room? Do the other four answer choices have too much space to wiggle or are they just completely and totally unsupported by the stim? What does "wiggle room" even mean and could someone provide an example without referring to PTs (as in, just make one up. All the examples on line are from PTs that I don't have with me right now)?

THANK YOU. I LOVE YOU ALL.

User avatar
crestor

Bronze
Posts: 313
Joined: Thu Jan 10, 2013 5:37 am

Re: "MSS has wiggle room." How much?

Post by crestor » Sat Sep 21, 2013 1:50 am

i really wonder what you look like irl scoobers

User avatar
Clearly

Gold
Posts: 4189
Joined: Sat Feb 11, 2012 4:09 pm

Re: "MSS has wiggle room." How much?

Post by Clearly » Sat Sep 21, 2013 1:52 am

Don't look too far into it. It still has a high burden of proof. Think of is as I must be able to infer this. Because of the nature of this question, you are generally going to like very weak answer choices, or answer choices that are just repeated in the stimulus. Also, aside from strong answers, be wary of answer choices are are inclusive of other answer choices. For instance if one answer is "Most of these are those" and another is "Some of these are those" one couldn't be right without the other also being right.

User avatar
crestor

Bronze
Posts: 313
Joined: Thu Jan 10, 2013 5:37 am

Re: "MSS has wiggle room." How much?

Post by crestor » Sat Sep 21, 2013 1:57 am

moreover, in terms of mss and mbt, i think an answer that is supported by the premise/conclusion relationship IE PLAUSIBLE CAN be the right answer on a soft must be true. the same sort of thinking can definitely not be applied to MBT.

for instance for alot of mbts, conditional relationships are involved where the answer must not only be supported but be visible in the relationship. it has to be much more than just supported but the choice must be "justified by one of the diagrammed statements or its contrapositive (BLUEPRINT)

Blueprint explicitly states "look for an answer choice that states the conclusion without exceeding the logical force of the stimulus" (lesson 2, page 19) for MSS

On the other hand, for MBT Blueprint says, "Anticipate an answer choice that follows from either the relationship between the statmenets in the stimulus or one of the stronger statements in isolation" (lesson 2, page 12)

I don't know how to explain it really. I think blueprint says it better.

User avatar
crestor

Bronze
Posts: 313
Joined: Thu Jan 10, 2013 5:37 am

Re: "MSS has wiggle room." How much?

Post by crestor » Sat Sep 21, 2013 1:59 am

Clearly wrote:Don't look too far into it. It still has a high burden of proof. Think of is as I must be able to infer this. Because of the nature of this question, you are generally going to like very weak answer choices, or answer choices that are just repeated in the stimulus. Also, aside from strong answers, be wary of answer choices are are inclusive of other answer choices. For instance if one answer is "Most of these are those" and another is "Some of these are those" one couldn't be right without the other also being right.

this. for MSS after doing 2000 or so total LR questions its just usually a much weaker choice. right on with the most of those and some of those trap answers. for instance if the question is how many beers are spoiled and the answers are most of the beers are spoiled and another is some of the beers are spoiled then the answer cannot be most of the beers because most automatically encompasses some in terms of percentages.


there was one question on a PT in the 20s though on a MSS where the most answer was right and the some answer was wrong and it was generally the same exact thing and that question still blows my mind.

Want to continue reading?

Register now to search topics and post comments!

Absolutely FREE!


Kimikho

Gold
Posts: 3971
Joined: Fri Mar 16, 2012 7:01 pm

Re: "MSS has wiggle room." How much?

Post by Kimikho » Sat Sep 21, 2013 2:46 am

Thank you! That was really helpful :). I'm looking back through my most recent MSS questions and definitely seeing a pattern with the inferences and not covering other answers, but I'm not quite seeing all of them restate the conclusion (such as the cat saliva one)...BUT I guess it makes sense that the answer would be a logical conclusion as in the sum of all of the parts in the stimulus.

But the most vs. some thing would cover that question anyways :).
crestor wrote:i really wonder what you look like irl scoobers
I've been told I'm a cross between Tina Fey, Dwight, and a hobbit.

bp shinners

Gold
Posts: 3086
Joined: Wed Mar 16, 2011 7:05 pm

Re: "MSS has wiggle room." How much?

Post by bp shinners » Mon Sep 23, 2013 1:16 pm

scoobers wrote:but I'm not quite seeing all of them restate the conclusion (such as the cat saliva one)...
That's only one thing a MSS question might do in a correct answer choice.

To get back to the original question, you're looking for something that you'd need to jump through hoops to explain away. So if the only way you can see that (A) couldn't be true based on the info in the stimulus is if every person in one class had exactly the same taste in art, and another class also had every person in it with a different from the first class but the same as each other taste in art, the answer is strongly supported because those are huge assumptions to be making.

User avatar
objection_your_honor

Silver
Posts: 625
Joined: Sat Feb 11, 2012 2:19 pm

Re: "MSS has wiggle room." How much?

Post by objection_your_honor » Mon Sep 23, 2013 1:29 pm

scoobers wrote: E.) Well, the stim says "reaching their highest population..." and that they are the main carrier of deer ticks...well then...
The mice reach their highest population density in these conditions, and they are the main carriers of Lyme disease bacteria (not of deer ticks). All we're given about deer ticks is that they're the transmission source to humans. We aren't given anything about the population relationship between mice and ticks.

I'm not sure if that's the answer you chose, but if so it looks like you just misread the stimulus a bit.

User avatar
Jeffort

Gold
Posts: 1888
Joined: Wed Jun 18, 2008 4:43 pm

Re: "MSS has wiggle room." How much?

Post by Jeffort » Tue Sep 24, 2013 12:16 am

bp shinners wrote:
scoobers wrote:but I'm not quite seeing all of them restate the conclusion (such as the cat saliva one)...
That's only one thing a MSS question might do in a correct answer choice.

To get back to the original question, you're looking for something that you'd need to jump through hoops to explain away.

So if the only way you can see that (A) couldn't be true based on the info in the stimulus is if every person in one class had exactly the same taste in art, and another class also had every person in it with a different from the first class but the same as each other taste in art, the answer is strongly supported because those are huge assumptions to be making.
This is a good way to look at it. ^

If you have to go as far as imagining a scenario that is very far fetched to the point of being extremely unlikely to happen in reality to conceive of a situation under which the answer choice could be false, you are looking at the correct answer. Somebody mentioned a good test method a while ago that is good for illustration. If you have to assume something that common sense tells you is absurd or extremely unlikely or that is contrary to basic everyday reality, such as aliens getting involved, in order to have circumstances where the CR could be false, you have the CR. Call it the aliens coming to earth test if you want.

However, the wiggle room occasionally is bigger than that.

I came across a recent question yesterday that has much more wiggle room than usual, and accordingly has pretty weak support. Based on the way I understand these questions in terms of burden of proof and the way I teach students to think about answers, I would have initially eliminated the credited answer under normal standards. I could only justify the credited answer by resorting to pointing out that it does have more support than the other answers, even though the support is really weak and far from making the answer anywhere close to air tight even without having to resort to an aliens invading earth scenario to see how it could be false.

Check out PT 61, second LR section (section 4), question #5, the Hamlet question.
The credited answer choice has a canyon of wiggle room!!! I think this is a really crappy question. It makes me suspicious that LSAC may not have as high of a burden of proof for credited MSS answers as us in the prep biz believe they do. For now I'm going to write this particular question off as an LSAC fluke/flawed question that shouldn't have made it through quality control but somehow did since otherwise I've going to have a really hard time explaining how it is consistent with other MSS questions where stretch answers like this are incorrect.

I'd love to hear others thoughts on this one, especially from other teachers/tutors. Perhaps I'll write LSAC about the question to see how they explain the support for the CR, this is the best case I've ever found of a question I believe is actually flawed.

Want to continue reading?

Register for access!

Did I mention it was FREE ?


User avatar
Clearly

Gold
Posts: 4189
Joined: Sat Feb 11, 2012 4:09 pm

Re: "MSS has wiggle room." How much?

Post by Clearly » Tue Sep 24, 2013 12:40 am

Jeffort wrote:
bp shinners wrote:
scoobers wrote:but I'm not quite seeing all of them restate the conclusion (such as the cat saliva one)...
That's only one thing a MSS question might do in a correct answer choice.

To get back to the original question, you're looking for something that you'd need to jump through hoops to explain away.

So if the only way you can see that (A) couldn't be true based on the info in the stimulus is if every person in one class had exactly the same taste in art, and another class also had every person in it with a different from the first class but the same as each other taste in art, the answer is strongly supported because those are huge assumptions to be making.
This is a good way to look at it. ^

If you have to go as far as imagining a scenario that is very far fetched to the point of being extremely unlikely to happen in reality to conceive of a situation under which the answer choice could be false, you are looking at the correct answer. Somebody mentioned a good test method a while ago that is good for illustration. If you have to assume something that common sense tells you is absurd or extremely unlikely or that is contrary to basic everyday reality, such as aliens getting involved, in order to have circumstances where the CR could be false, you have the CR. Call it the aliens coming to earth test if you want.

However, the wiggle room occasionally is bigger than that.

I came across a recent question yesterday that has much more wiggle room than usual, and accordingly has pretty weak support. Based on the way I understand these questions in terms of burden of proof and the way I teach students to think about answers, I would have initially eliminated the credited answer under normal standards. I could only justify the credited answer by resorting to pointing out that it does have more support than the other answers, even though the support is really weak and far from making the answer anywhere close to air tight even without having to resort to an aliens invading earth scenario to see how it could be false.

Check out PT 61, second LR section (section 4), question #5, the Hamlet question.
The credited answer choice has a canyon of wiggle room!!! I think this is a really crappy question. It makes me suspicious that LSAC may not have as high of a burden of proof for credited MSS answers as us in the prep biz believe they do. For now I'm going to write this particular question off as an LSAC fluke/flawed question that shouldn't have made it through quality control but somehow did since otherwise I've going to have a really hard time explaining how it is consistent with other MSS questions where stretch answers like this are incorrect.

I'd love to hear others thoughts on this one, especially from other teachers/tutors. Perhaps I'll write LSAC about the question to see how they explain the support for the CR, this is the best case I've ever found of a question I believe is actually flawed.
This really should have been a resolve the paradox question. I agree it sucks as a MSS, but would be pretty good for a paradox phrased answer choice. Calling it flawed is still a stretch to me. It does break from tradition of having a must be true element, but the phrase most strongly supported effectively means it just has to be a better answer than the other four answers, and in this case this is one plausible scenario that could follow from this, while the other choices aren't. Agreed, shitty question...at the least, shitty question stem choice.

Daily_Double

Silver
Posts: 1031
Joined: Tue Dec 04, 2012 8:45 pm

Re: "MSS has wiggle room." How much?

Post by Daily_Double » Tue Sep 24, 2013 12:58 am

Jeffort wrote:Check out PT 61, second LR section (section 4), question #5, the Hamlet question.
The credited answer choice has a canyon of wiggle room!!! I think this is a really crappy question. It makes me suspicious that LSAC may not have as high of a burden of proof for MSS questions as us in the prep biz believe they do. For now I'm going to write this particular question off as an LSAC fluke/flawed question that shouldn't have made it through quality control but somehow did since otherwise I've going to have a really hard time explaining how it is consistent with other MSS questions where stretch answers like this are incorrect.

I'd love to hear others thoughts on this one, especially from other teachers/tutors.
I should preface this by saying that I truly believe MSS stimuli imply something, that thing, either directly or to a lesser extent, is the correct answer. So let's see what the stimulus implies here.

Scholars are puzzled... (We know this will come up again because every single time a view is introduced in LR as incomplete, it is either completed/strenghtened or contradicted by an answer choice.)

Nice, we have a couple of facts:

(1) He did not possess a copy of the original --- Poor guy, it's an interesting read based upon my memory of it from high school, though I could be confusing it with King Lear.

(2) The restatement contains an accurate description of only one of the characters' speeches

So what's implied? Obviously we're looking for the author of the restatement since that's usually the point an answer to this argument structure touches upon. We know the author didn't write it, since the author would have possessed an original copy (or at least known some of the other speeches). Thus, the writer of the restatement of Hamlet wasn't Shakespeare.

But who else could it be? Well if it's not Shakespeare, the writer, it was probably an actor or a spectator of the original, assuming only one writer wrote each play. Shakespeare probably wouldn't have trusted Hamlet with anyone else anyways, because every moment he had to work by himself, or with others, on the play was time away from Anne Hathaway. So now we're down to an actor or spectator, being as spectators see all things, literally, assuming the wine didn't interfere with their vision, it's unlikely that a spectator would have a very accurate rendering of only one character.

Thus, we're left with an actor.

I'll concede that this question, in general, is a bit vague, but the stimulus doesn't lead to anything but this inference. I'll further concede that as soon as I started reading the stimulus, I knew the answer, so a fair amount of bias could be factored in. Though, I think I've made a decent argument for the correct answer. In addition, LSAC completely covers their bases by asking for the most strongly supported answer, implying some sort of relative amount of correctness. However, it is always the case that only one answer is supported (if not the most strongly supported by the argument) while the rest are unsupported and/or contradicted.

In this case we at least have an answer which is supported by the argument, since the others are not supported, it must be true that this is the most strongly supported.

But I'd be interested to hear why you think it's flawed. I'm always looking for alternative views to my own.

User avatar
Jeffort

Gold
Posts: 1888
Joined: Wed Jun 18, 2008 4:43 pm

Re: "MSS has wiggle room." How much?

Post by Jeffort » Tue Sep 24, 2013 2:35 am

I agree with your thoughts DD. My thoughts about it being flawed do not overcome the CYA nature of the MSS Q type/stem so I agree with your position that we cannot say it is per se a flawed question based on the stem and other answers.

My problem with it is in comparison to other MSS questions in terms of trying gauge how much wiggle room LSAC actually allows in terms of required burden of proof for the credited answer. There are plenty of MSS questions with incorrect answer choices that have stronger support in the stimulus than this stimulus provides for the answer that are nevertheless incorrect. Of course with those, there is always a better answer that is closer to being airtight.

This one just bothers me because the support is not very solid the way it is with most MSS questions. If the answer was probabilistic rather than definite I would have no issues with it. However, concluding with certainly that it WAS written by an actor is a larger leap in logical force than other MSS questions allow for the CR since the absolute nature of the statement is easily assailable as something that could possibly be false even if everything in the stimulus is true.

There is a palpable amount of uncertainty the evidence leaves open for other possibilities that makes saying it was definitely written by an actor something that could reasonably be false even given the evidence. As OP inquired about, it would be nice to know just how much wiggle room LSAC really allows with this question type in terms of evaluating 'could this be false in light of the evidence?' for elimination purposes. If there is a fixed minimum burden of proof regarding how airtight/unassailable an answer must be to qualify as the CR, it would be nice to have a clearer idea of where the fuzzy line is drawn rather than it being a moving target without a semi-clear minimum standard of support for logical certainty.

Most specifically, my problem with the answer choice is that the logical force is too strong compared to the supporting evidence since it only makes it likely, not really close to certain that the author was an actor. Combine that with the fact that LSAC commonly includes incorrect trap answers in many MSS questions that you can eliminate because the AC states something with a stronger degree of logical force than the related evidence establishes. Many attractive trap answers are constructed simply by taking something from the stimulus and restating it with the logical force stepped up a notch compared to the force of the evidence.

It just seems like a fairness and consistency issue. I guess if I really want to argue that it is a flawed LSAT question that should be tossed, I could focus on the meaning of the phrase 'strongly supported' since it does imply that the support must actually be at least strong to a degree vs being weakly supported and say that while somewhat supported, this answer choice isn't actually STRONGLY supported, but then we have to split hairs about where the line is between the top of being weakly supported and the beginning of being strongly supported. It is arguable, but I don't think it would be a big stretch to say this answer really isn't strongly supported, and instead is only weakly supported. Maybe if I frame a question to LSAC that way I might get them to address the issue a little. Hmmm.

EDIT: arrg, just looked at the question again. They didn't use the word STRONGLY in this stem, unlike most MSS questions! So much for my entire line of argument. :(

Alright everyone, critical announcement, there is a new, but not so new LR question type that has been lurking in our midst this whole time causing students to second guess themselves and argue with their instructors!! The MS-most supported type! Ohh jeeze, right now I do hate you LSAC. This little detail in the stem probably is the only important thing to pay attention to when worrying about wiggle room when you encounter an MSS (or actually MS!) Q type since removal of the word STRONGLY totally changes the standard for burden of proof leaving much more wiggle room/uncertainty. I cannot believe I haven't noticed this slightly different type of Q stem before. Hey dudes, all the big prep companies might have to re-write portions of their LR books now to include this rare Q type variation! I wonder how often that form of the stem has been used and if it is a new thing. I'll have to look closely to see if this has been going on for years right under my nose without me seeing it.

User avatar
neprep

Silver
Posts: 1066
Joined: Fri Jul 26, 2013 11:16 pm

Re: "MSS has wiggle room." How much?

Post by neprep » Tue Sep 24, 2013 4:47 am

Jeffort wrote:I agree with your thoughts DD. My thoughts about it being flawed do not overcome the CYA nature of the MSS Q type/stem so I agree with your position that we cannot say it is per se a flawed question based on the stem and other answers.

My problem with it is in comparison to other MSS questions in terms of trying gauge how much wiggle room LSAC actually allows in terms of required burden of proof for the credited answer. There are plenty of MSS questions with incorrect answer choices that have stronger support in the stimulus than this stimulus provides for the answer that are nevertheless incorrect. Of course with those, there is always a better answer that is closer to being airtight.

This one just bothers me because the support is not very solid the way it is with most MSS questions. If the answer was probabilistic rather than definite I would have no issues with it. However, concluding with certainly that it WAS written by an actor is a larger leap in logical force than other MSS questions allow for the CR since the absolute nature of the statement is easily assailable as something that could possibly be false even if everything in the stimulus is true.

There is a palpable amount of uncertainty the evidence leaves open for other possibilities that makes saying it was definitely written by an actor something that could reasonably be false even given the evidence. As OP inquired about, it would be nice to know just how much wiggle room LSAC really allows with this question type in terms of evaluating 'could this be false in light of the evidence?' for elimination purposes. If there is a fixed minimum burden of proof regarding how airtight/unassailable an answer must be to qualify as the CR, it would be nice to have a clearer idea of where the fuzzy line is drawn rather than it being a moving target without a semi-clear minimum standard of support for logical certainty.

Most specifically, my problem with the answer choice is that the logical force is too strong compared to the supporting evidence since it only makes it likely, not really close to certain that the author was an actor. Combine that with the fact that LSAC commonly includes incorrect trap answers in many MSS questions that you can eliminate because the AC states something with a stronger degree of logical force than the related evidence establishes. Many attractive trap answers are constructed simply by taking something from the stimulus and restating it with the logical force stepped up a notch compared to the force of the evidence.

It just seems like a fairness and consistency issue. I guess if I really want to argue that it is a flawed LSAT question that should be tossed, I could focus on the meaning of the phrase 'strongly supported' since it does imply that the support must actually be at least strong to a degree vs being weakly supported and say that while somewhat supported, this answer choice isn't actually STRONGLY supported, but then we have to split hairs about where the line is between the top of being weakly supported and the beginning of being strongly supported. It is arguable, but I don't think it would be a big stretch to say this answer really isn't strongly supported, and instead is only weakly supported. Maybe if I frame a question to LSAC that way I might get them to address the issue a little. Hmmm.

EDIT: arrg, just looked at the question again. They didn't use the word STRONGLY in this stem, unlike most MSS questions! So much for my entire line of argument. :(

Alright everyone, critical announcement, there is a new, but not so new LR question type that has been lurking in our midst this whole time causing students to second guess themselves and argue with their instructors!! The MS-most supported type! Ohh jeeze, right now I do hate you LSAC. This little detail in the stem probably is the only important thing to pay attention to when worrying about wiggle room when you encounter an MSS (or actually MS!) Q type since removal of the word STRONGLY totally changes the standard for burden of proof leaving much more wiggle room/uncertainty. I cannot believe I haven't noticed this slightly different type of Q stem before. Hey dudes, all the big prep companies might have to re-write portions of their LR books now to include this rare Q type variation! I wonder how often that form of the stem has been used and if it is a new thing. I'll have to look closely to see if this has been going on for years right under my nose without me seeing it.

But even then, Jeffort, isn't "most strongly supported" always explicitly or implicitly qualified by "which one of the following"? So, what the stem is asking is: of the answer choices listed, which one is most strongly supported, NOT which answer choice is by itself strongly supported. Even in this new "MS-Most Supported" type, the correct answer is "most strongly supported" of the answer choices presented, because it is most supported and the others are not. Thoughts?

I guess my question stems from my doubts about the importance of the adverb "strongly": For example, is there any difference between the following question stems:

Which one of the following appetizers is most highly appetizing?

v/s

Which one of the following appetizers is most appetizing?

Not doubting your analysis at all. Just curious. :)

bp shinners

Gold
Posts: 3086
Joined: Wed Mar 16, 2011 7:05 pm

Re: "MSS has wiggle room." How much?

Post by bp shinners » Wed Sep 25, 2013 6:57 pm

neprep wrote:But even then, Jeffort, isn't "most strongly supported" always explicitly or implicitly qualified by "which one of the following"? So, what the stem is asking is: of the answer choices listed, which one is most strongly supported, NOT which answer choice is by itself strongly supported. Even in this new "MS-Most Supported" type, the correct answer is "most strongly supported" of the answer choices presented, because it is most supported and the others are not. Thoughts?

I guess my question stems from my doubts about the importance of the adverb "strongly": For example, is there any difference between the following question stems:

Which one of the following appetizers is most highly appetizing?

v/s

Which one of the following appetizers is most appetizing?

Not doubting your analysis at all. Just curious. :)
It's more about test conventions than what the words literally mean or can imply. Each section says you should pick the best answer. While that aligns with just one answer being correct, it also leaves open multiple correct answers with one being "more correct." The latter, however, doesn't happen in practice (or, if it does happen, it happens rarely and causes controversy) - the test convention tells you more than the language in the instructions. So while your thoughts are correct, they don't pan out for almost every MSS question.

After looking at this, I tend to agree that MS questions might start cropping up more often, and that the burden for a correct answer is lower than MSS - at least, it is in this case. I might even lump these in with Resolve/Explain questions, as Clearly suggested. Or it might just be a fluke, and instead of throwing the question out, they changed the prompt. Only time will tell.

Register now!

Resources to assist law school applicants, students & graduates.

It's still FREE!


Post Reply

Return to “LSAT Prep and Discussion Forum”